Skip to Main Content

PrepTest 78, Game 2, Question 11

Transcript

This is the final question of PT 78 Game number 2. And it's a global question asking, which one of the following students cannot be assigned to 1922? Now, the best way to solve this kind of question asking for who can't be assigned to a particular slot. Is to look back at prior work, and usually you're gonna have examples of people who could go in that spot.

So if you have any examples of people who could go in 1922, based on other work. You could probably just get rid of those answers here. Sometimes you'll just be left with a single answer choice which means that's got to be correct. Other times you'll be left with two or three answer choices, and then you'll just try each of those answer choices out.

But assuming that we don't have any prior work, regarding who could be assigned to 1922. We just have to go down the line and try each answer choice. Let's take a look at answer choice A, could L be assigned in 1922? So if L goes in 1922, that means the third spot is gonna have to be a T. And based on rule number three, if T is in, that means R has to go in.

So I'm gonna put an R somewhere within group, based on rule four R brings 0 immediately before it. So I have to have an OR block like this, and you know what? I think we've stumbled on our answer because there's no room to put OR. R can't go in 1 cuz there's no room for O before it. And R can't go in 4 cuz there's no room for O before it, there's a T occupying 3.

So that means L can't go in 1922, A is gonna be our answer. In a time situation if I just went through this work like this, and I've proven that L can't go in 1922. I'm gonna circle A and I'm gonna move on, I'm not gonna check the other answer choices. But since this is review let's go ahead and see why the other ones could go, in 1922.

Answer choice B says M, you could put M in 1922. You could make a 1 a Y, and then you could make number 3L. And then you can make number 4 and O, this doesn't violate any rules. So M could go in 1921, or sorry, 1922. What about answer choice C, could O go in 1922? Well, I could put an O over here, and then I could put an M over here and 1.

And then I could put a Y in 4, and then I could make this one an L. So this does not violate any rules M O L Y, that's why answer choice C is not correct. And let me just go ahead and put my markings up again, remember we proved that L couldn't go 1922. We've proven that B and C, M and O could join in 1922.

Now, I think answer choice C is gonna be a popular answer choice here because. Sometimes you might think that O automatically brings R with it. But if you're reading rule number four carefully, it's not saying that anytime you put O in, you have to put R immediately after O. That's not what the fourth rule is saying. It's saying that anytime you put R in, then you have to put O immediately before it.

But O is allowed to go in without R. So that's why this situation does work. O is allowed to go in without R, and that's why O could go in 1922. Answer choice D says R, and I think this one's pretty obvious. There were plenty of times where you put R and 2, you put O and 1. And then you could put anything else in 4, you just make this one Y, and then this will work.

So R can go in 1922. Finally E says Y in 1922, could this work? Well, let me put Y in 2, M in 1. I'll put an O and 4, and then I could put an L and 3. This doesn't violate any rules, that's why Y could go in 1922

Read full transcript